Bluemo's Brain

Search

Search IconIcon to open search

負のlnの問題

Last updated Unknown Edit Source

    これは何でしょうか?

    • $\mathrm{Log}(-1)$の値の問題?
      • ですblu3mo.icon
      • 偏角の値域を$\theta \in (-\pi, \pi]$に制限すれば、$\mathrm{Log} z$は一意に定まるので問題ではないと思うのですがtakker.icon
    • $\mathrm{Log}(x) <0$となるような$x$の問題?
      • よく考えたら$x < 1$で常に成立するからこんなの当たり前だった